Pagina 1 di 1

Ingrandendo i fattori...

Inviato: 16 gen 2009, 10:23
da g(n)
Non so se sia già stato postato, comunque lo metto lo stesso. Dati $ a_1,\ldots,a_n $ reali positivi tali che $ a_1\cdot \ldots\cdot a_n=1 $, dimostrare che

$ (1+a_1)\cdot \ldots\cdot (1+a_n)\geq 2^n $

Buon lavoro :D

Inviato: 16 gen 2009, 14:11
da Davide90
La disuguaglianza di Chebycheff è valida anche per varie n-uple di reali? Cioè, è vero che (arrivando per esempio fino alla j-esima n-upla), con qualche condizione sull'ordinamento delle n-uple,
$ $ \displaystile n^{j-1}\cdot \sum_{i=1}^{n} a_i \cdot b_i \cdot c_i \cdots j_i \le \sum_{i=1}^{n} a_i \cdot \sum_{i=1}^{n} b_i \cdot \sum_{i=1}^{n} c_i \cdots \sum_{i=1}^{n} j_i $ :?:
Se è vero, allora è sufficiente considerare le n coppie $ \{ 1;a_i\} $ e la disuguaglianza è verificata. :roll:

Inviato: 16 gen 2009, 14:52
da Davide90
Molto probabilmente quella scritta sopra è una scemenza... Dunque provo a scrivere una soluzione corretta :)
$ $ \displaystile (1+a_1)\cdot \ldots\cdot (1+a_n) = 1+\sum_{sym}a_1 +\sum_{sym} a_1a_2+\sum_{sym}a_1a_2a_3+\dots+\prod_{i=1}^na_i $ ma per AM-QM
$ $ \displaystile \geq 1+\binom{n}{1} \left(\prod_{1=1}^n a_i\right)^{\frac{\binom{n}{0}}{\binom{n}{1}}} +\binom{n}{2} \left(\prod_{1=1}^n a_i\right)^{\frac{\binom{n}{1}}{\binom{n}{2}}} +\binom{n}{3} \left(\prod_{1=1}^n a_i\right)^{\frac{\binom{n}{2}}{\binom{n}{3}}}+\dots +\binom{n}{n} \left(\prod_{1=1}^n a_i\right)^{\frac{\binom{n}{n}}{\binom{n}{n}}} = 1+\binom{n}{2} +\binom{n}{3} +\dots +\binom{n}{n}= 2^n $
All'ultimo passaggio ho usato la formula per la somma dei coefficienti binomiali)
(che fatica... :shock: ) Bel problema però!! :D

Inviato: 16 gen 2009, 15:10
da SkZ
sbaglio o manca un $ $\sum a_i $?
cmq mi pare cambi nulla

Inviato: 16 gen 2009, 15:35
da Davide90
Si, giusto... :oops: Ora correggo

Inviato: 16 gen 2009, 17:25
da g(n)
Sì, giusto :D

Volevo proporre una mini generalizzazione:

$ (m+a_1)\cdot \ldots \cdot(m+a_n)\geq (m+1)^n $

ma dovrebbe venire automatica a questo punto.

Inviato: 16 gen 2009, 18:07
da Jacobi
Davide90 ha scritto:Molto probabilmente quella scritta sopra è una scemenza... Dunque provo a scrivere una soluzione corretta :)
$ $ \displaystile (1+a_1)\cdot \ldots\cdot (1+a_n) = 1+\sum_{sym}a_1 +\sum_{sym} a_1a_2+\sum_{sym}a_1a_2a_3+\dots+\prod_{i=1}^na_i $ ma per AM-QM
$ $ \displaystile \geq 1+\binom{n}{1} \left(\prod_{1=1}^n a_i\right)^{\frac{\binom{n}{0}}{\binom{n}{1}}} +\binom{n}{2} \left(\prod_{1=1}^n a_i\right)^{\frac{\binom{n}{1}}{\binom{n}{2}}} +\binom{n}{3} \left(\prod_{1=1}^n a_i\right)^{\frac{\binom{n}{2}}{\binom{n}{3}}}+\dots +\binom{n}{n} \left(\prod_{1=1}^n a_i\right)^{\frac{\binom{n}{n}}{\binom{n}{n}}} = 1+\binom{n}{2} +\binom{n}{3} +\dots +\binom{n}{n}= 2^n $
All'ultimo passaggio ho usato la formula per la somma dei coefficienti binomiali)
(che fatica... :shock: ) Bel problema però!! :D
c'e' un modo mooolto piu semplice (:D) x farlo: per AM-GM e' $ 1+a_i \geq 2\sqrt{ 1 \cdot a_i} \longrightarrow \prod_{i=1}^{n}{(1+a_i)} \geq \prod_{i=1}^{n}{2 \sqrt{a_i}} = 2^n \sqrt{\prod_{i=1}^{n}{a_i}} = 2^n $ (si puo fare lo stesso anke con l'altra)

Inviato: 16 gen 2009, 23:37
da g(n)
Volendo si può svolgere il prodotto, vedere che saltano fuori 2^n addendi, applicare AM-GM e, per simmetria, ogni $ a_i $ comparirà lo stesso numero di volte nella GM, cosicchè GM=1, da cui la tesi.

@Jacobi: :o ottima soluzione, non ci avevo pensato :D

Inviato: 15 apr 2009, 22:27
da Carlein
Si lo so che rasento la necrofilia di topic anch'io. Niente di profondo e veramente interessante(molto probabilmente alla fin fine è quasi la stessa pappa delle cose che avete detto ma servita in un altro modo), ma così per sfizio m'andava di scrivere, e metto come ho fatto io.
Se uno sviluppa la somma come Davide può accoppiare ad ognuno dei $ 2^n $ addendi il prodotto che dà l'inverso (insomma se uno ha messo $ a_1....a_j $ ora lo accoppia con $ a_{j+1}....a_n $) ora è noto che(ma due conti bastano a mostrarlo) $ x+1/x \geq 2 $,x positivo. Dunque sia S quella somma, abbiamo $ S \geq 2^{n-1}2 $$ =2^n $
:oops: